QUESTIONS
DIRECTIONS: Choose the one best response to each question.
I-1. Which of the following is the best definition of evidence-based medicine?
A. A summary of existing data from existing clinical trials with a critical methodological review and statistical analysis of summative data
B. A type of research that compares the results of one approach to treating a disease with another approach to treating the same disease
C. Clinical decision making support tools developed by professional organizations that include expert opinions and data from clinical trials
D. Clinical decision making supported by data, preferably from randomized controlled clinical trials
E. One physician’s clinical experience in caring for multiple patients with a specific disorder over many years
I-2. All of the following are part of the informed consent process EXCEPT:
A. Alternatives and likely consequences of the alternatives to the procedure
B. Ascertainment of understanding by the patient
C. Discussion of the details of the procedure
D. Outlining the patient’s wishes if he or she becomes unable to make decisions
E. Risks and benefits of the procedure
I-3. Which of the following is the standard measure for determining the impact of a health condition on a population?
A. Disability-adjusted life years
B. Infant mortality
C. Life expectancy
D. Standardized mortality ratio
E. Years of life lost
I-4. In high-income countries, what category of disease accounts for the greatest percentage of disability-adjusted life years lost?
A. Alcohol abuse
B. Chronic obstructive pulmonary disease
C. Diabetes mellitus
D. Ischemic heart disease
E. Unipolar depressive disorders
I-5. What is the leading cause of death in low-income countries?
A. Diarrheal diseases
B. Human immunodeficiency virus
D. Ischemic heart disease
D. Lower respiratory disease
E. Malaria
I-6. You are working with the public health minister of Malawi in a project to decrease malarial deaths in children younger than 5 years of age. All of the following strategies are part of the World Health Organization Roll Back Malaria plan EXCEPT:
A. Artemisinin-based combination therapy
B. Early treatment with chloroquine alone
C. Indoor residual spraying
D. Insecticide-treated bed nets
E. Intermittent preventive treatment during pregnancy
I-7. A 38-year-old woman is evaluated for chest pain. She has no risk factors for coronary artery disease, but a stress test is ordered by a physician in the emergency department. You are called for a cardiology consult when an exercise ECG stress test result is positive. You estimate that the pretest probability of coronary artery disease is 10% and determine that this is most likely a false-positive stress test with a low posttest probability of coronary artery disease. This is an example of which of the following principles used in medical decision making?
A. Bayes’ theorem
B. High positive predictive value
C. High specificity
D. Low negative predictive value
E. Low sensitivity
I-8. A new diagnostic test for predicting latent tuberculosis is introduced into clinical practice. In clinical trials, it was determined to have a sensitivity of 90% and a specificity of 80%. A specific clinical population of 1000 individuals has a prevalence of tuberculosis of 10%. How many individuals with latent tuberculosis would be correctly identified in this population?
A. 10
B. 80
C. 90
D. 100
E. 180
I-9. In the above scenario, how many individuals would be erroneously told they have latent tuberculosis?
A. 10
B. 90
C. 180
D. 720
E. 900
I-10. A receiver operating characteristic (ROC) curve is constructed for a new test for disease X. All of the following statements regarding the ROC curve are true EXCEPT:
A. One criticism of the ROC curve is that it is developed for testing only one test or clinical parameter with exclusion of other potentially relevant data.
B. The ROC curve allows the selection of a threshold value for a test that yields the best sensitivity with the fewest false-positive test results.
C. The axes of the ROC curve are sensitivity versus 1 – specificity.
D. The ideal ROC curve would have a value of 0.5.
E. The value of the ROC curve is calculated as the area under the curve generated from the true-positive rate versus the false-positive rate.
I-11. Which of the following values is affected by the disease prevalence in a population?
A. Number needed to treat
B. Positive likelihood ratio
C. Positive predictive value
D. Sensitivity
E. Specificity
I-12. Drug X is investigated in a meta-analysis for its effect on mortality after a myocardial infarction. It is found that mortality drops from 10 to 2% when this drug is administered. What is the absolute risk reduction conferred by drug X?
A. 2%
B. 8%
C. 20%
D. 200%
E. None of the above
I-13. How many patients will have to be treated with drug X to prevent one death?
A. 2
B. 8
C. 12.5
D. 50
E. 93
I-14. When considering a potential screening test, what endpoints should be considered to assess the potential gain from a proposed intervention?
A. Absolute and relative impact of screening on the disease outcome
B. Cost per life year saved
C. Increase in the average life expectancy for the entire population
D. Number of subjects screened to alter the outcome in one individual
E. All of the above
I-15. A 55-year-old man who smokes cigarettes is enrolled in a lung cancer screening trial based on performance of yearly CT scans over a period of 5 years. At year 2, he is found to have a 2-cm right lower lobe lung nodule that is a non–small cell lung cancer upon surgical removal. At that time, there were no positive lymph nodes. The cancer recurs, and the patient subsequently dies from lung cancer 6 years after his initial diagnosis. A person with a similar smoking history who is not participating in the trial is discovered to have a 3-cm lung nodule that is also non–small cell lung cancer. Upon surgical resection, one lymph node is positive. This person also dies from lung cancer after a period of 3 years. What conclusion can be made about the use of the CT screening for lung cancer in these patients?
A. CT screening for lung cancer improves mortality in smokers.
B. It is unable to be determined if CT screening for lung cancer led to any difference in survival because one cannot determine if lag time bias is present.
C. It is unable to be determined if CT screening for lung cancer led to any difference in survival because one cannot determine if lead time bias is present.
D. Selection bias may cause apparent differences in survival in this trial, and one should be cautious in making conclusions with regards to CT screening for lung cancer.
E. The radiation received as part of the CT scan screening led to lung cancer in the initial patient and contributed to the first patient’s overall mortality.
I-16. According to the U.S. Preventive Services Task Force, what is the recommended screening interval for thyroid disease in women older than the age of 30 years?
A. Every 5 years beginning at age 30 years
B. Once at age 30 years
C. Once at age 30 years and again in 10 years if the test result is normal
D. Periodically
E. There is no recommended screening for thyroid disease recommended by the U.S. Preventive Services Task Force
I-17. Which preventative intervention leads to the largest average increase in life expectancy for a target population?
A. A regular exercise program for a 40-year-old man
B. Getting a 35-year-old smoker to quit smoking
C. Mammography in women age 50–70 years
D. Pap smears in women age 18–65 years
E. Prostate-specific antigen (PSA) and digital rectal examination for a man older than 50 years old
I-18. All of the following patients should receive a lipid screening profile EXCEPT:
A. A 16-year-old boy with type 1 diabetes
B. A 17-year-old female teen who recently began smoking
C. A 23-year-old healthy man who is starting his first job
D. A 48-year-old woman beginning menopause
E. A 62-year-old man with no past medical history
I-19. A 43-year-old woman is diagnosed with pulmonary blastomycosis and is initiated on therapy with oral itraconazole therapy. All of the following could affect the bio-availability of this drug EXCEPT:
A. Coadministration with a cola beverage
B. Coadministration with oral contraceptive pills
C. Formulation of the drug (liquid vs. capsule)
D. pH of the stomach
E. Presence of food in the stomach
I-20. A 24-year-old woman with cystic fibrosis is admitted to the hospital with an exacerbation. She is known to be colonized with Pseudomonas aeruginosa and is started on intravenous therapy with cefepime 1 g IV every 8 hours and tobramycin 10 mg/kg IV once daily. You want to ensure that the risk of nephrotoxicity is low. When should the tobramycin level be checked?
A. 30 minutes after the first dose
B. 2 hours after the first dose
C. 2 hours before second dose
D. Immediately before the fourth dose
E. There is no need to check drug levels if the patient has normal renal function
I-21. A 68-year-old man with ischemic cardiomyopathy has been treated with digoxin 250 μg daily for the past year. He has chronic kidney disease with a stable baseline creatinine of 2.1 mg/dL. He is initiated on an oral amiodarone load for new-onset atrial fibrillation with rapid ventricular response. Over 1 week, he develops increasing nausea, vomiting, and fatigue. On presentation to the emergency department, he is lethargic and difficult to arouse with a heart rate of 45 beats/min and a blood pressure of 88/50 mmHg. His laboratory values demonstrate a potassium of 5.2 meq/L, creatinine of 3.0 mg/dL, and a digoxin level of 13 ng/mL. His ECG shows complete heart block. What is the most appropriate treatment for this patient?
A. Digitalis-specific antibody (Fab) fragments alone
B. Digitalis-specific antibody fragments plus hemodialysis
C. Digitalis-specific antibody fragments plus hemoperfusion
D. Plasmapheresis alone
E. Volume resuscitation and observation
I-22. A 48-year-old woman with a generalized seizure disorder has been taking phenytoin for the past 10 years with good control of her disease. She also has a history of hepatitis C virus infection acquired via a blood transfusion received after an automobile accident in her teens. She currently takes phenytoin 100 mg tid, lactulose 30 g tid, and spironolactone 25 mg daily. She is brought to the emergency department by her husband, who reports that she has had increasing lethargy for the past week. On examination, her blood pressure is 100/60 mmHg, heart rate is 88 beats/min, respiratory rate is 20 breaths/min, and oxygen saturation is 98% on room air. She is afebrile. She is minimally responsive to voice and follows no commands. There is no nuchal rigidity. Her abdomen is distended with a positive fluid wave but without tenderness. She has spider angiomata, caput medusa, and palmar erythema. She does not appear to have asterixis. She does have horizontal nystagmus on examination. Her laboratory values include Na, 134 meq/L; potassium, 3.9 meq/L; chloride, 104 meq/L; and bicarbonate, 20 meq/L. Creatinine is 1.0 mg/dL. The white blood cell count is 10,000/μL with a normal differential. Her liver function tests are unchanged from baseline with the exception of an albumin that is now 2.1 g/dL compared with 3 months ago when her level was 2.9 g/dL. Ammonia level is 15 μmol/L, and her phenytoin level is 17 mg/L. A paracentesis shows a white blood cell count of 100/μL that is 80% neutrophils. What test would be most likely to demonstrate the cause of the patient’s change in mental status?
A. CT scan of the head
B. Electroencephalogram (EEG)
C. Free phenytoin level
D. Gram stain of ascites fluid
E. Gram stain of cerebrospinal fluid (CSF)
I-23. A 55-year-old Japanese woman is found to have a 3-cm mass in the right lower lobe of the lung. She is a lifelong nonsmoker. The mass is positive on positron emission tomography scan as are contralateral and ipsilateral lymph nodes in the mediastinum. A biopsy demonstrates the mass to be a moderately differentiated adenocarcinoma, and a left hilar lymph node also demonstrates adenocarcinoma. Clinically, this places the patient as a stage IIIB non–small cell lung cancer, and the patient and her oncologist decide to treat with chemotherapy. Molecular testing demonstrates an exon 19 deletion in the tyrosine kinase domain of the epidermal growth factor receptor and no mutation in k-ras. What is the best choice for initial chemotherapy in this patient?
A. Carboplatin plus paclitaxel
B. Carboplatin and paclitaxel plus erlotinib
C. Docetaxel alone
D. Erlotinib alone
E. Gemcitabine plus docetaxel
I-24. A 26-year-old woman received an allogeneic bone marrow transplant 9 months ago for acute myelogenous leukemia. Her transplant course is complicated by graft-versus-host disease with diarrhea, weight loss, and skin rash. She is immunosuppressed with tacrolimus 1 mg bid and prednisone 7.5 mg daily. She recently was admitted to the hospital with shortness of breath and fevers to 101.5°F. She has a chest CT showing nodular pneumonia, and fungal organisms are seen on a transbronchial lung biopsy. The culture demonstrates Aspergillus fumigatus, and a serum galactomannan level is elevated. She is initiated on therapy with voriconazole 6 mg/kg IV every 12 hours for 1 day, decreasing to 4 mg/kg IV every 12 hours beginning on day 2. Two days after starting voriconazole, she is no longer febrile but is complaining of headaches and tremors. Her blood pressure is 150/92 mmHg, up from 108/60 mmHg on admission. On examination, she has developed 1+ pitting edema in the lower extremities. Her creatinine has risen to 1.7 mg/dL from 0.8 mg/dL on admission. What is the most likely cause of the patient’s current clinical picture?
A. Aspergillus meningitis
B. Congestive heart failure
C. Recurrent graft-versus-host disease
D. Tacrolimus toxicity
E. Thrombotic thrombocytopenic purpura caused by voriconazole
I-25. A 45-year-old man is diagnosed with primary syphilis after development of a penile ulcer. Results of a rapid plasma reagin and fluorescent treponemal antibody absorption tests are both positive. He is treated with benzathine penicillin G 2.4 million units intramuscularly as a one-time dose. Ten days after the injection, the patient presents to the emergency department complaining of fevers, rash, and diffuse joint pains with muscle aches. On physical examination, the patient has a temperature of 38.3°F, heart rate of 110 beats/min, and blood pressure of 112/76 mmHg. His HEENT, chest, cardiovascular, and abdominal examination findings are normal. He has an urticarial rash on trunk, back, and extremities. There is swelling and warmth of the knees, wrists, and metacarpophalangeal joints bilaterally. In addition, there is pain with palpation of the tendinous insertions of the Achilles tendons and patellar tendons bilaterally. The penile ulcer has a dry base and has decreased in size compared with previously. Laboratory studies show a white cell count of 10,100/μL (80% neutrophils, 15% lymphocytes, 3% monocytes, and 2% eosinophils). The erythrocyte sedimentation rate is 55 seconds. Antinuclear antibodies and rheumatoid factor results are negative. A urethral swab is negative for Chlamydia trachomatis and Neisseria gonorrhea. What is the most likely diagnosis?
A. Disseminated gonococcal infection
B. Inadequate treatment of secondary syphilis
C. Jarisch-Herxheimer reaction
D. Seronegative rheumatoid arthritis
E. Serum sickness caused by benzathine penicillin
I-26. Which of the following classes of medicines has been linked to the occurrence of hip fractures in elderly adults?
A. Benzodiazepines
B. Opiates
C. Angiotensin-converting enzyme inhibitors
D. Beta-blockers
E. Atypical antipsychotics
I-27. Patients taking which of the following drugs should be advised to avoid drinking grapefruit juice?
A. Amoxicillin
B. Aspirin
C. Atorvastatin
D. Prevacid
E. Sildenafil
I-28. Which of the following diseases is responsible for a greater percentage of deaths in women compared with men?
A. Alzheimer’s disease
B. Cerebrovascular disease
C. Chronic obstructive pulmonary disease
D. Sepsis
E. All of the above
I-29. Which of the following statements regarding coronary heart disease (CHD) in women when compared with men is TRUE?
A. Angina is a rare symptom in women with CHD.
B. At the time of diagnosis of CHD, women typically have fewer comorbidities compared with men.
C. Physicians are less likely to consider CHD in women and are also less likely to recommend both diagnostic and therapeutic procedures in women.
D. Women and men present with CHD at similar ages.
E. Women are more likely to present with ventricular tachycardia, but men more commonly have cardiac arrest or cardiogenic shock.
I-30. Which of the following is an independent risk factor for coronary heart disease in women but not men?
A. Elevated total triglyceride levels
B. Hypertension
C. Low high-density lipoprotein cholesterol
D. Obesity
E. Smoking
I-31. All of the following diseases are more common in women than men EXCEPT:
A. Depression
B. Hypertension
C. Obesity
D. Rheumatoid arthritis
E. Type 1 diabetes mellitus
I-32. Which of the following statements regarding Alzheimer’s disease and gender are true?
A. Alzheimer’s disease affects men and women at equal rates.
B. Alzheimer’s disease affects men two times more commonly than women.
C. In a recent placebo-controlled trial, postmenopausal hormone therapy did not show improvement in disease progression in women with Alzheimer’s disease.
D. The difference in deaths from Alzheimer’s disease between men and women can be entirely accounted for by the difference in life expectancy between men and women.
E. Women with Alzheimer’s disease have higher levels of circulating estrogen than women without Alzheimer’s disease.
I-33. All of the following are changes in the cardiovascular system seen in pregnancy EXCEPT:
A. Decreased blood pressure
B. Increased cardiac output
C. Increased heart rate
D. Increased plasma volume
E. Increased systemic vascular resistance
I-34. A 36-year-old woman has a history of hypertension and is planning on starting a family. She is currently taking lisinopril 10 mg daily for control of her blood pressure. She wants to stop taking her oral contraceptive medications. Her current blood pressure is 128/83 mmHg. What do you advise her about ongoing treatment with antihypertensive medications?
A. Because the cardiovascular changes that occur during pregnancy lead to a fall in blood pressure, she can safely discontinue her lisinopril when she stops her oral contraceptives.
B. She should continue lisinopril and start hydrochlorothiazide.
C. She should discontinue lisinopril and start irbesartan.
D. She should discontinue lisinopril and start labetalol.
E. She should not get pregnant because she is high risk of complications.
I-35. Which of the following cardiovascular conditions is a contraindication to pregnancy?
A. Atrial septal defect without Eisenmenger syndrome
B. Idiopathic pulmonary arterial hypertension
C. Marfan syndrome
D. Mitral regurgitation
E. Prior peripartum cardiomyopathy with a current ejection fraction of 65%
I-36. A 33-year-old woman with diabetes mellitus and hypertension presents to the hospital with seizures during week 37 of her pregnancy. Her blood pressure is 156/92 mmHg. She has 4+ proteinuria. Management should include all of the following EXCEPT:
A. Emergent delivery
B. Intravenous labetalol
C. Intravenous magnesium sulfate
D. Intravenous phenytoin
I-37. A 27-year-old woman develops left leg swelling during week 20 of her pregnancy. Left lower extremity ultrasonogram reveals a left iliac vein deep venous thrombosis (DVT). Proper management includes:
A. Bedrest
B. Catheter-directed thrombolysis
C. Enoxaparin
D. Inferior vena cava filter placement
E. Warfarin
I-38. In which of the following categories should women undergo routine screening for gestational diabetes?
A. Age greater than 25 years
B. Body mass index greater than 25 kg/m2
C. Family history of diabetes mellitus in a first-degree relative
D. African American
E. All of the above
I-39. Which of the following surgeries would be considered at the greatest risk for postsurgical complications?
A. Carotid endarterectomy
B. Non-emergent repair of a thoracic aortic aneurysm
C. Resection of a 5-cm lung cancer
D. Total colectomy for colon cancer
E. Total hip replacement
I-40. A 64-year-old man is contemplating undergoing elective cholecystectomy for biliary colic and cholelithiasis. He has a history of coronary artery disease with coronary artery bypass surgery performed at the age of 51 after an anterior wall myocardial infarction. His most recent ejection fraction 2 years previously was 35%. He also has a 45 pack-year history of tobacco, quitting after his surgery 13 years previously. Since his bypass surgery, he reports failure to return to full functional capacity. You ask him about his current exercise capacity. Which of the following would be considered poor exercise tolerance and increase his risk of perioperative complications?
A. Inability to achieve 4 metabolic equivalents during an exercise test
B. Inability to carry 15–20 lb
C. Inability to climb two flights of stairs at a normal pace
D. Inability to walk four blocks at a normal pace
E. All of the above
I-41. A 74-year-old man is scheduled to undergo total colectomy for recurrent life-threatening diverticular bleeding. He denies any chest pain with exertion but is limited in his physical activity because of degenerative arthritis of his knees. He has no history of coronary artery disease or congestive heart failure but does have diabetes mellitus and hypertension. His current medications include aspirin 81 mg daily, atorvastatin 10 mg daily, enalapril 20 mg daily, and insulin glargine 25 units daily in combination with insulin lispro on a sliding scale. His blood pressure is 128/86 mmHg. His physical examination findings are normal. His most recent hemoglobin A1C is 6.3%, and his creatinine is 1.5 mg/dL. You elect to perform an electrocardiogram preoperatively, and it demonstrates Q waves in leads II, III, and aVF. Based on this information, what is his expected his postoperative risk of a major cardiac event?
A. 0.5%
B. 1%
C. 5%
D. 10%
E. 20%
I-42. All of the following are risk factors for postoperative pulmonary complications EXCEPT:
A. Age greater than 60 years
B. Asthma with a peak expiratory flow rate of 220 L/min
C. Chronic obstructive pulmonary disease
D. Congestive heart failure
E. Forced expiratory volume in 1 second of 1.5 L
I-43. You are caring for a 56-year-old woman who was admitted to the hospital with a change in mental status. She underwent a right-sided mastectomy and axillary lymph node dissection 3 years previously for stage IIIB ductal carcinoma. Serum calcium is elevated at 15.3 mg/dL. A chest radiograph demonstrates innumerable pulmonary nodules, and a head CT shows a brain mass in the right frontal lobe with surrounding edema. Despite correcting her calcium and treating cerebral edema, the patient remains confused. You approach the family to discuss the diagnosis of widely metastatic disease and the patient’s poor prognosis. Which of the following is NOT a component of the seven elements for communicating bad news (P-SPIKES approach)?
A. Assess the family’s perception of her current illness and the status of her underlying cancer diagnosis.
B. Empathize with the family’s feelings and provide emotional support.
C. Prepare mentally for the discussion.
D. Provide an appropriate setting for discussion.
E. Schedule a follow-up meeting in 1 day to reassess whether there are additional informational and emotional needs.
I-44. Which of the following is not a component of a living will?
A. Delineation of specific interventions that would be acceptable to the patient under certain conditions
B. Description of values that should guide discussions regarding terminal care
C. Designation of a health care proxy
D. General statements regarding whether the patient desires receipt of life-sustaining interventions such as mechanical ventilation
I-45. A 72-year-old woman has stage IV ovarian cancer with diffuse peritoneal studding. She is developing increasing pain in her abdomen and is admitted to the hospital for pain control. She previously was treated with oxycodone 10 mg orally every 6 hours as needed. Upon admission, she is initiated on morphine intravenously via patient-controlled analgesia. During the first 48 hours of her hospitalization, she received an average daily dose of morphine 90 mg and reports adequate pain control unless she is walking. What is the most appropriate opioid regimen for transitioning this patient to oral pain medication?
I-46. You are asked to consult on 62-year-old man who was recently found to have newly metastatic disease. He was originally diagnosed with cancer of the prostate 5 years previously and presented to the hospital with back pain and weakness. Magnetic resonance imaging (MRI) demonstrated bony metastases to his L2 and L5 vertebrae with spinal cord compression at the L2 level only. On bone scan images, there was evidence of widespread bony metastases. He has been started on radiation and hormonal therapy, and his disease has shown some response. However, he has become quite depressed since the metastatic disease was found. His family reports that he is sleeping for 18 or more hours daily and has stopped eating. His weight is down 12 lb over 4 weeks. He expresses profound fatigue, hopelessness, and a feeling of sadness. He claims to have no interest in his usual activities and no longer interacts with his grandchildren. What is the best approach to treating this patient’s depression?
A. Do not initiate pharmacologic therapy because the patient is experiencing an appropriate reaction to his newly diagnosed metastatic disease.
B. Initiate therapy with doxepin 75 mg nightly.
C. Initiate therapy with fluoxetine 10 mg daily.
D. Initiate therapy with fluoxetine 10 mg daily and methylphenidate 2.5 mg twice daily in the morning and at noon.
E. Initiate therapy with methylphenidate 2.5 mg twice daily in the morning and at noon.
I-47. You are treating a 76-year-old woman with Alzheimer’s disease admitted to the intensive care unit for aspiration pneumonia. After 7 days of mechanical ventilation, her family requests that care be withdrawn. The patient is palliated with fentanyl intravenously at a rate of 25 μg/hr and midazolam intravenously at 2 mg/hr. You are urgently called to the bedside 15 minutes after the patient is extubated because the patient’s daughter is distraught. She states that you are “drowning” her mother and is upset because her mother appears to be struggling to breathe. When you enter the room, you hear a gurgling noise that is coming from accumulated secretions in the oropharynx. You suction the patient for liberal amounts of thin salivary secretions and reassure the daughter that you will make her mother as comfortable as possible. Which of the following interventions may help with the treatment of the patient’s oral secretions?
A. Increased infusion rate of fentanyl
B. N-acetylcysteine nebulized
C. Pilocarpine drops
D. Placement of a nasal trumpet and oral airway to allow easier access for aggressive suctioning
E. Scopolamine patches
I-48. Which of the following is the most common type of preventable adverse event in hospitalized patients?
A. Adverse drug events
B. Diagnostic failures
C. Falls
D. Technical complications of procedures
E. Wound infections
I-49. All of the following statements regarding the use of complementary and alternative medicine (CAM) in the US are true EXCEPT:
A. Acupuncture is the most frequently used CAM approach in the US
B. CAM approaches represent approximately 10% of out-of-pocket medical expenses in the US
C. Control of back or musculoskeletal pain is a common reason for US patients to utilize CAM approaches
D. Recent estimates suggest 30-40% of Americans use CAM approaches
E. The most common reasons US patients seek CAM approaches is for management of symptoms poorly controlled by conventional approaches
I-50. Independent of insurance status, income, age, and comorbid conditions, African American patients are less likely to receive equivalent levels of care compared with white patients for the following scenarios:
A. Prescription of analgesic for pain control
B. Referral to renal transplantation
C. Surgical treatment for lung cancer
D. Utilization of cardiac diagnostic and therapeutic procedures
E. All of the above
I-51. All of the following statements regarding the difference between breast cancer in pregnant versus nonpregnant women are true EXCEPT:
A. Estrogen-positive tumors are more common in pregnant women.
B. Her-2 positivity is more common in pregnant women.
C. A higher stage is more common in pregnant women.
D. Positive lymph nodes are more common in pregnant women.
E. Tumor size at diagnosis is larger in pregnant women.
I-52. A 32-year-old woman seeks evaluation for cough that has been present for 4 months. She reports that the cough is present day and night. It does awaken her from sleep and is worse in the early morning hours. She also notes the cough to be worse in cold weather and after exercise. She describes the cough as dry and has no associated shortness of breath or wheezing. She gives no antecedent history of an upper respiratory tract infection that preceded the onset of cough. She has a medical history of pulmonary embolus occurring in the postpartum period 6 years previously. Her only medication is norgestimate/ethinyl estradiol. She works as an elementary school teacher. On review of systems, she reports intermittent itchy eyes and runny nose that is worse in the spring and fall. She denies postnasal drip and heartburn. Her physical examination findings are normal with the exception of coughing when breathing through an open mouth. A chest radiograph is also normal. Spirometry demonstrates a forced expiratory volume in 1 second (FEV1) of 3.0 L (85% predicted), forced vital capacity (FVC) of 3.75 L (88% predicted), and FEV1/FVC ratio of 80%. After administration of a bronchodilator, the FEV1increases to 3.3 L (10% change). What would you recommend next in the evaluation and treatment of this patient?
A. Initiate a nasal corticosteroid.
B. Initiate a proton pump inhibitor.
C. Perform a methacholine challenge test.
D. Perform a nasopharyngeal culture for Bordetella pertussis.
E. Reassure the patient that there are no pulmonary abnormalities and continue supportive care.
I-53. A 56-year-old man presents to his primary care physician complaining of coughing up blood. He has felt ill for the past 4 days with a low-grade fever and cough. The cough was initially productive of yellow-green sputum, but it now is sputum mixed with red blood. He estimates that he has produced about 1–2 tsp (5–10 mL) of blood in the past day. He smokes 1 pack of cigarettes daily and has done so since the 15 years of age. He is known to have moderate chronic obstructive pulmonary disease and coronary artery disease. He takes aspirin, metoprolol, lisinopril, tiotropium, and albuterol as needed. His physical examination is notable for a temperature of 37.8°C (100.0°F). Bilateral expiratory wheezing and coarse rhonchi are heard on examination. Chest radiograph is normal. What is the most likely cause of hemoptysis in this individual?
A. Acute bronchitis
B. Infection with tuberculosis
C. Lung abscess
D. Lung cancer
E. Medications
I-54. A 65-year-old man with a known squamous cell carcinoma near the right upper lobe bronchus is admitted to intensive care after coughing up more than 100 mL of bright red blood. He appears in significant respiratory distress with an oxygen saturation of 78% on room air. He continues to have violent coughing with ongoing hemoptysis. He had a prior pulmonary embolus and is being treated with warfarin. His last INR was therapeutic at 2.5 three days previously. All of the following would be useful in the immediate management of this patient EXCEPT:
A. Consultation with anesthesia for placement of a dual-lumen endotracheal tube.
B. Consultation with interventional radiology for embolization.
C. Consultation with thoracic surgery for urgent surgical intervention if conservative management fails.
D. Correction of the patient’s coagulopathy.
E. Positioning of the patient in the left lateral decubitus position.
I-55. Microbial agents have been used as bioweapons since ancient times. All of the following are key features of microbial agents that are used as bioweapons EXCEPT:
A. Environmental stability
B. High morbidity and mortality rates
C. Lack of rapid diagnostic capability
D. Lack of readily available antibiotic treatment
E. Lack of universally available and effective vaccine
I-56. Ten individuals in Arizona are hospitalized over a 4-week period with fever and rapidly enlarging and painful lymph nodes. Seven of these individuals experience severe sepsis, and three die. While reviewing the epidemiologic characteristics of these individuals, you note that they are all illegal immigrants and have recently stayed in the same immigrant camp. Blood cultures are growing gram-negative rods that are identified as Yersinia pestis. You notify local public health officials and the Centers for Disease Control and Prevention. Which of the following factors indicate that this is NOT likely to be an act of bioterrorism?
A. The area affected was limited to a small immigrant camp.
B. The individuals presented with symptoms of bubonic plague rather than pneumonic plague.
C. The individuals were in close contact with one another, suggesting possible person-to-person transmission.
D. The mortality rate was less than 50%.
E. Yersinia pestis is not environmentally stable for longer than 1 hour.
I-57. Which of the following routes of dispersal are likely for botulinum toxin used as a bioweapon?
A. Aerosol
B. Contamination of the food supply
C. Contamination of the water supply
D. A and B
E. All of the above
I-58. Anthrax spores can remain dormant in the respiratory tract for how long?
A. 1 week
B. 6 weeks
C. 6 months
D. 1 year
E. 3 years
I-59. Twenty recent attendees at a National Football League game arrive at the emergency department complaining of shortness of breath, fever, and malaise. Chest radiographs show mediastinal widening on several of these patients, prompting a concern for inhalational anthrax as a result of a bioterror attack. Antibiotics are initiated, and the Centers for Disease Control and Prevention is notified. What form of isolation should be instituted for these patients in the hospital?
A. Airborne
B. Contact
C. Droplet
D. None
I-60. The Centers for Disease Control and Prevention (CDC) has designated several biologic agents as category A in their ability to be used as bioweapons. Category A agents include agents that can be easily disseminated or transmitted, result in high mortality, can cause public panic, and require special action for public health preparedness. All of the following agents are considered category A EXCEPT:
A. Bacillus anthracis
B. Francisella tularensis
C. Ricin toxin from Ricinus communis
D. Smallpox
E. Yersinia pestis
I-61. All of the following chemical agents of bioterrorism are correctly identified by their mechanism of injury EXCEPT:
A. Arsine—asphyxiant
B. Chlorine gas—pulmonary damage
C. Cyanogen chloride—nerve agent
D. Mustard gas—vesicant
E. Sarin—nerve agent
I-62. Over the course of 12 hours, 24 individuals present to a single emergency department complaining of a sunburn-like reaction with development of large blisters. Most of these individuals are also experiencing irritation of the eyes, nose, and pharynx. Two individuals developed progressive dyspnea, severe cough, and stridor requiring endotracheal intubation. On physical examination, all of the patients exhibited conjunctivitis and nasal congestion. Erythema of the skin was greatest in the axillae, neck, and antecubital fossae. Many of the affected had large, thin-walled bullae on the extremities that were filled with a clear or straw-colored fluid. On further questioning, all of the affected individuals had been shopping at a local mall within the past 24 hours and ate at the food court. Many commented on a strong odor of burning garlic in the food court at that time. You suspect a bioterrorism act. Which of the following is TRUE with regard to the likely agent causing the patients’ symptoms?
A. 2-Pralidoxime should be administered to all affected individuals.
B. The associated mortality rate of this agent is more than 50%.
C. The cause of respiratory distress in affected individuals is related to direct alveolar injury and adult respiratory distress syndrome.
D. The erythema that occurs can be delayed as long as 2 days after exposure and depends on several factors, including ambient temperature and humidity.
E. The fluid within the bullae should be treated as a hazardous substance that can lead to local reactions and blistering with exposure.
I-63. A 24-year-old man is evaluated immediately after exposure to chlorine gas as an act of chemical terrorism. He currently denies dyspnea. His respiratory rate is 16 breaths/min and oxygen saturation is 97% on room air. All of the following should be included in the immediate treatment of this individual EXCEPT:
A. Aggressive bathing of all exposed skin areas
B. Flushing of the eyes with water or normal saline
C. Forced rest and fresh air
D. Immediate removal of clothing if no frostbite
E. Maintenance of a semiupright position
I-64. You are a physician working in an urban emergency department when several patients are brought in after the release of an unknown gas at the performance of a symphony. You are evaluating a 52-year-old woman who is not able to talk clearly because of excessive salivation and rhinorrhea, although she is able to tell you that she feels as if she lost her sight immediately upon exposure. At present, she also has nausea, vomiting, diarrhea, and muscle twitching. On physical examination, the patient has a blood pressure of 156/92 mmHg, a heart rate of 92, a respiratory rate of 30 breaths/min, and a temperature of 37.4°C (99.3°F). She has pinpoint pupils with profuse rhinorrhea and salivation. She also is coughing profusely, with production of copious amounts of clear secretions. A lung examination reveals wheezing on expiration in bilateral lung fields. The patient has a regular rate and rhythm with normal heart sounds. Bowel sounds are hyperactive, but the abdomen is not tender. She is having diffuse fasciculations. At the end of your examination, the patient abruptly develops tonic-clonic seizures. Which of the following agents is most likely to cause this patient’s symptoms?
A. Arsine
B. Cyanogen chloride
C. Nitrogen mustard
D. Sarin
E. VX
I-65. All the following should be used in the treatment of this patient EXCEPT:
A. Atropine
B. Decontamination
C. Diazepam
D. Phenytoin
E. 2-Pralidoxime chloride
I-66. All of the following statements are true regarding the results of detonation of a low-yield nuclear device by a terror group EXCEPT:
A. After recovery of initial exposure symptoms, the patient remains at risk of systemic illness for up to 6 weeks.
B. Appropriate medical therapy can change the LD50 from approximately 4–8 gray (Gy).
C. Initial mortality is mostly caused by shock blast and thermal damage.
D. Most of the total mortality is related to release of alpha and beta particles.
E. The hematopoietic, gastrointestinal, and neurologic systems are most likely involved in acute radiation syndrome.
I-67. A “dirty” bomb is detonated in downtown Boston. The bomb was composed of cesium-137 with trinitrotoluene. In the immediate aftermath, an estimated 30 people were killed because of the power of the blast. The fallout area was about 0.5 mile, with radiation exposure of about 1.8 Gy. An estimated 5000 people have been potentially exposed to beta and gamma radiation. Most of these individuals show no sign of any injury, but about 60 people have evidence of thermal injury. What is the most appropriate approach to treating the injured victims?
A. All individuals who have been exposed should be treated with potassium iodide.
B. All individuals who have been exposed should be treated with Prussian blue.
C. All individuals should be decontaminated before transportation to the nearest medical center for emergency care to prevent exposure of health care workers.
D. Severely injured individuals should be transported to the hospital for emergency care after removing the victims’ clothes because the risk of exposure to health care workers is low.
E. With this degree of radiation exposure, no further testing and treatment are needed.
I-68. A 37-year-old woman is brought to the ICU after her elective laparoscopic cholecystectomy is complicated by a temperature of 105°F, tachycardia, and systemic hypotension. Examination is notable for diffuse muscular rigidity. Which of the following drugs should be administered immediately?
A. Acetaminophen
B. Dantrolene
C. Haloperidol
D. Hydrocortisone
E. Ibuprofen
I-69. Hyperthermia is defined as:
A. A core temperature greater than 40.0°C
B. A core temperature greater than 41.5°C
C. An uncontrolled increase in body temperature despite a normal hypothalamic temperature setting
D. An elevated temperature that normalizes with antipyretic therapy
E. Temperature greater than 40.0°C, rigidity, and autonomic dysregulation
I-70. Which of the following conditions is associated with increased susceptibility to heat stroke in elderly adults?
A. A heat wave
B. Antiparkinsonian therapy
C. Bedridden status
D. Diuretic therapy
E. All of the above
I-71. A recent 18-year-old immigrant from Kenya presents to a university clinic with fever, nasal congestion, severe fatigue, and a rash. The rash started with discrete lesions at the hairline that coalesced as the rash spread caudally. There is sparing of the palms and soles. Small white spots with a surrounding red halo are noted on examination of the palate. The patient is at risk for developing which of the following in the future?
A. Encephalitis
B. Epiglottitis
C. Opportunistic infections
D. Postherpetic neuralgia
E. Splenic rupture
I-72. A 23-year-old woman with a chronic lower extremity ulcer related to prior trauma presents with rash, hypotension, and fever. She has had no recent travel or outdoor exposure and is up to date on all of her vaccinations. She does not use IV drugs. On examination, the ulcer looks clean with a well-granulated base and no erythema, warmth, or pustular discharge. However, the patient does have diffuse erythema that is most prominent on her palms, conjunctiva, and oral mucosa. Other than profound hypotension and tachycardia, the remainder of the examination is nonfocal. Laboratory results are notable for a creatinine of 2.8 mg/dL, aspartate aminotransferase of 250 U/L, alanine aminotransferase of 328 U/L, total bilirubin of 3.2 mg/dL, direct bilirubin of 0.5 mg/dL, INR of 1.5, activated partial thromboplastin time of 1.6 × control, and platelets at 94,000/μL. Ferritin is 1300 μg/mL. The patient is started on broad-spectrum antibiotics after appropriate blood cultures are drawn and is resuscitated with IV fluid and vasopressors. Her blood cultures are negative at 72 hours; at this point, her fingertips start to desquamate. What is the most likely diagnosis?
A. Juvenile rheumatoid arthritis (JRA)
B. Leptospirosis
C. Staphylococcal toxic shock syndrome
D. Streptococcal toxic shock syndrome
E. Typhoid fever
I-73. A 75-year-old man with chronic systolic heart failure requiring high-dose diuretics and lisinopril is seen by his primary care physician for acute onset of right great toe pain with redness and swelling. He is unable to bear weight on this foot. On examination, he is afebrile and has normal vital signs. His complaints in his right great toe are verified. No other joints are involved, and he appears otherwise to be in well-compensated heart failure. Prednisone and allopurinol are prescribed. Five days later, the patient is seen in the emergency department with a temperature of 101°F and a rash throughout his body and mouth. On examination, he has diffuse erythema, areas of skin exfoliation, and oral and orbital edema. Mucous membranes are not involved. Laboratory studies show mild transaminitis and peripheral eosinophilia. Which of the following syndromes describes this condition?
A. Acute bacterial endocarditis
B. Angioedema caused by lisinopril
C. Drug-induced hypersensitivity syndrome caused by allopurinol
D. MRSA cellulitis
E. Staphylococcal toxic shock syndrome caused by septic arthritis
I-74. A 50-year-old man is evaluated for fevers and weight loss of uncertain etiology. He first developed symptoms 3 months previously. He reports daily fevers to as high as 39.4°C (103°F) with night sweats and fatigue. Over this same period, his appetite has been decreased, and he has lost 50 lb compared with his weight at his last annual examination. Fevers have been documented in his primary care physician’s office to as high as 38.7°C (101.7°F). He has no exposures or ill contacts. His medical history is significant for diabetes mellitus, obesity, and obstructive sleep apnea. He is taking insulin glargine 50 U daily. He works in a warehouse driving a forklift. He has not traveled outside of his home area in a rural part of Virginia. He has never received a blood transfusion and is married with one female sexual partner for the past 25 years. On examination, no focal findings are identified. Multiple laboratory studies have been performed that have shown nonspecific findings only with exception of an elevated calcium at 11.2 g/dL. A complete blood count showed a white blood cell count of 15,700/μL with 80% polymorphonuclear cells, 15% lymphocytes, 3% eosinophils, and 2% monocytes. The peripheral smear is normal. The hematocrit is 34.7%. His erythrocyte sedimentation rate (ESR) is elevated at 57 mm/hr. A rheumatologic panel is normal, and the ferritin is 521 ng/mL. Liver and kidney function are normal. The serum protein electrophoresis demonstrated polyclonal gammopathy. HIV, Epstein-Barr virus (EBV), and cytomegalovirus (CMV) testing are negative. The urine Histoplasma antigen result is negative. Routine blood cultures for bacteria, chest radiograph, and purified protein derivative (PPD) testing results are negative. A CT scan of the chest, abdomen, and pelvis has borderline enlargement of lymph nodes in the abdomen and retroperitoneum to 1.2 cm. What would be the next best step in determining the etiology of fever in this patient?
A. Empiric treatment with corticosteroids
B. Empiric treatment for Mycobacterium tuberculosis
C. Needle biopsy of enlarged lymph nodes
D. PET-CT imaging
E. Serum angiotensin-converting enzyme levels
I-75. A 48-year-old man is brought to the emergency department (ED) in January after being found unresponsive in a city park. He has alcoholism and was last seen by his daughter about 12 hours before being brought to the ED. At that time, he left their home intoxicated and agitated. He left seeking additional alcohol because his daughter had poured out his last bottle of vodka hoping that he would seek treatment. On presentation, he has a core body temperature of 88.5°F (31.4°C), heart rate of 48 beats/min, respiratory rate of 28 breaths/min, and blood pressure of 88/44 mmHg; oxygen saturation is unable to be obtained. The arterial blood gas demonstrates a pH of 7.05, PaCO2of 32 mmHg, and PaO2 of 56 mmHg. Initial blood chemistries demonstrate a sodium of 132 meq/L, potassium of 5.2 meq/L, chloride of 94 meq/L, bicarbonate of 10 meq/L, blood urea nitrogen of 56 mg/dL, and creatinine of 1.8 mg/dL. Serum glucose is 63 mg/dL. The serum ethanol level is 65 mg/dL. The measured osmolality is 328 mOsm/kg. ECG demonstrates sinus bradycardia with a long first-degree atrioventricular block and J waves. In addition to initiating a rewarming protocol, what additional tests should be performed in this patient?
A. Endotracheal intubation with hyperventilation to a goal PaCO2 of less than 20 mmHg
B. Intravenous hydration with a 1–2 L bolus of warmed lactated Ringer’s solution
C. No other measures are necessary because interpretation of the acid–base status is unreliable with this degree of hypothermia.
D. Measure levels of ethylene glycol and methanol
E. Placement of a transvenous cardiac pacemaker
I-76. A homeless man is evaluated in the emergency department. He has noted that after he slept outside during a particularly cold night his left foot has become clumsy and feels “dead.” On examination, the foot has hemorrhagic vesicles distributed throughout the foot distal to the ankle. The foot is cool and has no sensation to pain or temperature. The right foot is hyperemic but does not have vesicles and has normal sensation. The remainder of the physical examination findings are normal. Which of the following statements regarding the management of this disorder is true?
A. Active foot rewarming should not be attempted.
B. During the period of rewarming, intense pain can be anticipated.
C. Heparin has been shown to improve outcomes in this disorder.
D. Immediate amputation is indicated.
E. Normal sensation is likely to return with rewarming.
I-77. A 25-year-old woman becomes lightheaded and experiences a syncopal event while having her blood drawn during a cholesterol screening. She has no medical history and takes no medications. She experiences a brief loss of consciousness for about 20 seconds. She has no seizure-like activity and immediately returns to her usual level of functioning. She is diagnosed with vasovagal syncope, and no follow-up testing is recommended. Which of the following statements regarding neurally mediated syncope is TRUE?
A. Neurally mediated syncope occurs when there are abnormalities of the autonomic nervous system.
B. Proximal and distal myoclonus do not occur during neurally mediated syncope and should increase the likelihood of a seizure.
C. The final pathway of neurally mediated syncope results in a surge of the sympathetic nervous system with inhibition of the parasympathetic nervous system.
D. The primary therapy for neurally mediated syncope is reassurance, avoidance of triggers, and plasma volume expansion.
E. The usual finding with cardiovascular monitoring is hypotension and tachycardia.
I-78. A 76-year-old woman is brought to the emergency department after a syncopal event that occurred while she was singing in her church choir. She has a history of hypertension, diabetes mellitus, and chronic kidney disease (stage III). She does recall at least two prior episodes of syncope similar to this one. Her medications include insulin glargine 40 units daily, lispro insulin sliding scale, lisinopril 20 mg daily, and hydrochlorothiazide 25 mg daily. By the time she arrived in the emergency department, she reports feeling back to her usual self. She does recall feeling somewhat lightheaded before the syncopal events but does not recall the event itself. Witnesses report some jerking of her upper extremities. She regained full consciousness in less than 2 minutes. Her current vital signs include blood pressure of 110/62 mmHg, heart rate of 84 beats/min, respiratory rate of 16 breaths/min, and oxygen saturation of 95% on room air. She is afebrile. Her physical examination is unremarkable and includes a normal neurologic examination. Which of the following would be least helpful in determining the etiology of the patient’s syncope?
A. CT scan of the head
B. Electrocardiogram
C. Fingerstick glucose measurement
D. Orthostatic blood pressure measurement
E. Tilt table testing
I-79. A 48-year-old man presents to the emergency department complaining of dizziness. He describes it as a sensation that the room is spinning. All of the following would be consistent with a central cause of vertigo EXCEPT:
A. Absence of tinnitus
B. Gaze-evoked nystagmus
C. Hiccups
D. Inhibition of nystagmus by visual fixation
E. Purely vertical nystagmus
I-80. A 62-year-old woman presents complaining of severe dizziness. She notes it especially when she turns over in bed and immediately upon standing. Her initial physical examination findings are normal. Upon further testing, you ask the patient to sit with her head turned 45 degrees to the right. You lower the patient to the supine position and extend the head backward 20 degrees. This maneuver immediately reproduces the patient’s symptoms, and you note torsional nystagmus. What is the most appropriate next step in evaluation and treatment of this patient?
A. MRI of the brainstem
B. Methylprednisolone taper beginning at 100 mg daily
C. Repositioning (Epley) maneuvers
D. Rizatriptan 10 mg orally once
E. Valacyclovir 1000 mg three times daily for 7 days
I-81. A 42-year-old man presents complaining of progressive weakness over a period of several months. He reports tripping over his toes while walking and has dropped a cup of hot coffee on one occasion because he felt too weak to continue to hold it. A disorder affecting lower motor neurons is suspected. All of the following findings would be found in an individual with a disease primarily affecting lower motor neurons EXCEPT:
A. Decreased muscle tone
B. Distal greater than proximal weakness
C. Fasciculations
D. Hyperactive tendon reflexes
E. Severe muscle atrophy
I-82. A 78-year-old man is seen in clinic because of recent falls. He reports gait difficulties with a sensation of being off balance at times. One recent fall caused a shoulder injury requiring surgery to repair a torn rotation cuff. In epidemiologic case series, what is the most common cause of gait disorders?
A. Cerebellar degeneration
B. Cerebrovascular disease with multiple infarcts
C. Cervical myelopathy
D. Parkinson’s disease
E. Sensory deficits
I-83. A 65-year-old man presents complaining of frequent falls and gait abnormalities. He first noticed the difficulty about 6 months ago. He has a history of hypertension and hypothyroidism and hyperlipidemia. His current medications include amlodipine 10 mg daily, simvastatin 20 mg daily, and levothyroxine 75 μg daily. On neurologic examination, you observe his gait to be wide based with short, shuffling steps. He has difficulty rising from his chair and initiating his gait. Upon turning, he takes multiple steps and appears unsteady. However, cerebellar testing results are normal, including heel-to-shin and Romberg testing. He has no evidence of sensory deficits in the lower extremities, and strength is 5/5 throughout all tested muscle groups. He shows no evidence of muscle spasticity on passive movement. His neurologic examination is consistent with which of the following causes?
A. Alcoholic cerebellar degeneration
B. Communicating hydrocephalus
C. Neurosyphilis
D. Multiple system atrophy
E. Lumbar myelopathy
I-84. A 74-year-old woman is admitted to the medical intensive care unit with confusion and sepsis from a urinary origin. Her initial blood pressure was 70/40 mmHg with a heart rate of 130 beats/min. She is volume resuscitated but requires dopamine to maintain an adequate blood pressure. Her mental status improved initially, but now she is agitated and pulling at her IV catheters. She is screaming that she is trapped, and she is not oriented to place or year. All of the following statements regarding the patient’s condition are true EXCEPT:
A. An episode of delirium is associated with an inhospital mortality rate of 25% to 33%.
B. A patient who has an episode of delirium in the hospital is more likely to be discharged to a nursing home.
C. Delirium is associated with an increased risk of all-cause mortality for at least 1 year after hospital discharge.
D. Delirium is typically short-lived and does not persist longer than several days.
E. Individuals who experience delirium have longer lengths of stay in the hospital.
I-85. You are covering the night shift at a local hospital and are called acutely to the bedside of a 62-year-old man to evaluate a change in his mental status. He was admitted 36 hours previously for treatment of community-acquired pneumonia. He received treatment with levofloxacin 500 mg daily and required oxygen 2 L/min. He has a medical history of tobacco abuse, diabetes mellitus, and hypertension. He reports alcohol intake of 2–4 beers daily. His vital signs at 10 PM were blood pressure of 138/85 mmHg, heart rate of 92 beats/min, respiratory rate of 20 breaths/min, temperature of 37.4°C (99.3°F), and SaO2of 92% on oxygen 2 L/min. Currently, the patient is agitated and pacing his room. He is reporting that he needs to leave the “meeting” immediately and go home. He states that if he does not do this, someone is going to take his house and car away. He has removed his IV and oxygen tubing from his nose. His last vital signs taken 30 minutes previously were blood pressure of 156/92 mmHg, heart rate of 118 beats/min, respiratory rate of 26 breaths/min, temperature of 38.3°C (100.9°F), and oxygen saturation of 87% on room air. He is noted to be somewhat tremulous and diaphoretic. All of the following should be considered as part of the patient’s diagnostic workup EXCEPT:
A. Arterial blood gas testing
B. Brain imaging with MRI or head CT
C. Fingerstick glucose testing
D. More thorough review of the patient’s alcohol intake with his wife
E. Review of the recent medications received by the patient
I-86. Delirium, an acute confusional state, is a common disorder that remains a major cause of morbidity and mortality in the United States. Which of the following patients is at the highest risk for developing delirium?
A. A 36-year-old man admitted to the medical ward with a deep venous thrombosis
B. A 55-year-old man postoperative day 2 from a total colectomy
C. A 68-year-old woman admitted to the intensive care unit (ICU) with esophageal rupture
D. A 74-year-old woman in the preoperative clinic before hip surgery
E. An 84-year-old man living in an assisted living facility
I-87. Which of the following is the most common finding in aphasic patients?
A. Alexia
B. Anomia
C. Comprehension
D. Fluency
E. Repetition
I-88. A 65-year-old man experiences an ischemic cerebrovascular accident affecting the territory of the right anterior cerebral artery. After the stroke, an assessment reveals the findings shown in Figure I-88. What diagnosis does this figure suggest?
A. Construction apraxia
B. Hemianopia
C. Hemineglect
D. Object agnosia
E. Simultanagnosia
FIGURE I-88
I-89. A 42-year-old man is evaluated for excessive sleepiness that is interfering with his ability to work. He works at a glass factory that requires him to work rotating shifts. He typically cycles across day (7 AM–3 PM), evening (3 PM–11 PM), and night (11 PM–7 AM) shifts over the course of 4 weeks. He notes the problem to be most severe when he is on the night shift. Twice he has fallen asleep on the job. Although no accidents have occurred, he has been threatened with loss of his job if he falls asleep again. His preferred sleep schedule is 10 PM until 6 AM, but even when he is working day shifts, he typically only sleeps from about 10:30 PM until 5:30 AM. However, he feels fully functional at work on day and evening shifts. After his night shifts, he states that he finds it difficult to sleep when he first gets home, frequently not falling asleep until 10 AM or later. He is up by about 3 PM when his children arrive home from school. He drinks about 2 cups of coffee daily but tries to avoid drinking more than this. He does not snore and has a body mass index of 21.3 kg/m2. All of the following are reasonable approaches to treatment in this man EXCEPT:
A. Avoidance of bright light in the morning after his shifts
B. Exercise in the early evening before going to work
C. Melatonin 3 mg taken at bedtime on the morning after a night shift
D. Modafinil 200 mg taken 30–60 minutes before starting a shift
E. Strategic napping of no more than 20 minutes during breaks at work
I-90. A 45-year-old woman presents for evaluation of abnormal sensations in her legs that keep her from sleeping at night. She first notices the symptoms around 8 PM when she is sitting quietly watching television. She describes the symptoms as “ants crawling in my veins.” Although the symptoms are not painful, they are very uncomfortable and worsen when she lies down at night. They interfere with her ability to fall asleep about four times weekly. If she gets out of bed to walk or rubs her legs, the symptoms disappear almost immediately only to recur as soon as she is still. She also sometimes takes a very hot bath to alleviate the symptoms. During sleep, her husband complains that she kicks him throughout the night. She has no history of neurologic or renal disease. She currently is perimenopausal and has been experiencing very heavy and prolonged menstrual cycles over the past several months. The physical examination findings, including thorough neurologic examination, are normal. Her hemoglobin is 9.8 g/dL and hematocrit is 30.1%. The mean corpuscular volume is 68 fL. Serum ferritin is 12 ng/mL. Which is the most appropriate initial therapy for this patient?
A. Carbidopa/levodopa
B. Hormone replacement therapy
C. Iron supplementation
D. Oxycodone
E. Pramipexole
I-91. A 20-year-old man presents for evaluation of excessive daytime somnolence. He is finding it increasingly difficult to stay awake during his classes. Recently, his grades have fallen because whenever he tries to read, he finds himself drifting off. He finds that his alertness is best after exercising or brief naps of 10–30 minutes. Because of this, he states that he takes 5 or 10 “catnaps” daily. The sleepiness persists despite averaging 9 hours of sleep nightly. In addition to excessive somnolence, he reports occasional hallucinations that occur as he is falling asleep. He describes these occurrences as a voice calling his name as he drifts off. Perhaps once weekly, he awakens from sleep but is unable to move for a period of about 30 seconds. He has never had apparent loss of consciousness but states that whenever he is laughing, he feels heaviness in his neck and arms. Once he had to lean against a wall to keep from falling down. He undergoes an overnight sleep study and multiple sleep latency test. There is no sleep apnea. His mean sleep latency on five naps is 2.3 minutes. In three of the five naps, rapid eye movement sleep is present. Which of the following findings of this patient is most specific for the diagnosis of narcolepsy?
A. Cataplexy
B. Excessive daytime somnolence
C. Hypnagogic hallucinations
D. Rapid eye movement sleep in more than two naps on a multiple sleep latency test
E. Sleep paralysis
I-92. Which of the following is the most common sleep disorder in the U.S. population?
A. Delayed sleep phase syndrome
B. Insomnia
C. Obstructive sleep apnea
D. Narcolepsy
E. Restless legs syndrome
I-93. In which stage of sleep are the parasomnias somnambulism and night terrors most likely to occur?
A. Stage 1
B. Stage 2
C. Slow-wave sleep
D. Rapid eye movement sleep
I-94. A 44-year-old man is seen in the emergency department after a motor vehicle accident. The patient says, “I never saw that car coming from the right side.” On physical examination, his pupils are equal and reactive to light. His visual acuity is normal; however, there are visual field defects in both eyes laterally (bitemporal hemianopia). Which of the following is most likely to be found on further evaluation?
A. Retinal detachment
B. Occipital lobe glioma
C. Optic nerve injury
D. Parietal lobe infarction
E. Pituitary adenoma
I-95. A 42-year-old construction worker complains of waking up with a red, painful left eye. She often works without goggles at her construction site. Her history is notable for hypertension, inflammatory bowel disease, diabetes, and prior IV drug use. Her only current medication is lisinopril. On examination, the left eye is diffusely red and sensitive to light. The eyelids are normal. In dim light, visual acuity is normal in both eyes. All of the following diagnoses will explain her findings EXCEPT:
A. Acute angle-closure glaucoma
B. Anterior uveitis
C. Corneal abrasion
D. Posterior uveitis
E. Transient ischemic attack
I-96. A 75-year-old triathlete complains of gradually worsening vision over the past year. It seems to be involving near and far vision. The patient has never required corrective lenses and has no significant medical history other than diet-controlled hypertension. He takes no regular medications. Physical examination is normal except for bilateral visual acuity of 20/100. There are no focal visual field defects and no redness of the eyes or eyelids. Which of the following is the most likely diagnosis?
A. Age-related macular degeneration
B. Blepharitis
C. Diabetic retinopathy
D. Episcleritis
E. Retinal detachment
I-97. All of the following statements regarding olfaction are true EXCEPT:
A. Decrements in olfaction may lead to nutritional deficiency.
B. More than 40% of patients with traumatic anosmia will regain normal function over time.
C. Significant decrements in olfaction are present in more than 50% of the population 80 years and older.
D. The most common identifiable cause of long-lasting or permanent loss of olfaction in outpatients is severe respiratory infection.
E. Women identify odorants better than men at all ages.
I-98. A 64-year-old man is evaluated for hearing loss that he thinks is worse in his left ear. His wife and children have told him for years that he does not listen to them. Recently, he has failed to hear the chime of the alarm on his digital watch, and he admits to focusing on the lips of individuals speaking to him because he sometimes has difficulties in word recognition. In addition, he reports a continuous buzzing that is louder in his left ear. He denies any sensation of vertigo, headaches, or balance difficulties. He has worked in a factory for many years that makes parts for airplanes, and the machinery that he works with sits to his left primarily. He has no family history of deafness, although his father had hearing loss as he aged. He has a medical history of hypertension, hyperlipidemia, and coronary artery disease. You suspect sensorineural hearing loss related to exposure to the intense noise in the factory for many decades. Which of the following findings would you expect on physical examination?
A. A deep tympanic retraction pocket seen above the pars flaccida on the tympanic membrane.
B. Cerumen impaction in the external auditory canal.
C. Hearing loss that is greater at lower frequencies on pure tone audiometry.
D. Increased intensity of sound when a tuning fork is placed on the mastoid process when compared with placement near the auditory canal.
E. Increased intensity of sound in the right ear when a tuning fork is placed in the midline of the forehead.
I-99. A 32-year-old woman presents to her primary care physician complaining of nasal congestion and drainage and headache. Her symptoms originally began about 7 days ago with rhinorrhea and sore throat. For the past 5 days, she has been having increasing feelings of fullness and pressure in the maxillary area that is causing her headaches. The pressure is worse when she bends over, and she also notices it while lying in bed at night. She is otherwise healthy and has not had fevers. On physical examination, there is purulent nasal drainage and pain with palpation over bilateral maxillary sinuses. What is the best approach to ongoing management of this patient?
A. Initiate therapy with amoxicillin 500 mg three times daily for 10 days.
B. Initiate therapy with levofloxacin 500 mg daily for 10 days.
C. Perform a sinus aspirate for culture and sensitivities.
D. Perform a sinus CT.
E. Treat with oral decongestants and nasal saline lavage.
I-100. A 28-year-old man seeks evaluation for sore throat for 2 days. He has not had a cough or rhinorrhea. He has no other medical conditions and works as a daycare provider. On examination, tonsillar hypertrophy with membranous exudate is present. What is the next step in the management of this patient?
A. Empiric treatment with amoxicillin 500 mg twice daily for 10 days
B. Rapid antigen detection test for Streptococcus pyogenes only
C. Rapid antigen detection test for Streptococcus pyogenes plus throat culture if the rapid test result is negative
D. Rapid antigen detection test for Streptococcus pyogenes plus a throat culture regardless of result
E. Throat culture only
I-101. A 62-year-old man presents to his physician complaining of shortness of breath. All of the following findings are consistent with left ventricular dysfunction as a cause of the patient’s dyspnea EXCEPT:
A. Feeling of chest tightness
B. Nocturnal dyspnea
C. Orthopnea
D. Pulsus paradoxus greater than 10 mmHg
E. Sensation of air hunger
I-102. A 42-year-old woman seeks evaluation for a cough that has been present for almost 3 months. The cough is mostly dry and non-productive, but occasionally productive of yellow phlegm. She reports that the cough is worse at night and often wakes her from sleep. She denies any recent upper respiratory tract infection, allergic rhinitis, fever, chills or cough. She recalls her mother told her that she had asthma as a child but she has never felt symptomatic wheezing as an adult. She exercises regularly but continues to smoke 1 pack per day of cigarettes; she’d like to quit. The patient takes no medications. Her physical examination is unremarkable. Which of the following is indicated at this point?
A. Chest PET-CT
B. Chest radiograph
C. Measurement of serum angiotensin-converting enzyme (ACE)
D. Measurement of serum IgE
E. Sinus CT
I-103. In the patient described above, her chest radiograph is normal and further history reveals a long history of symptoms suggestive of GERD. She also admits that her cough is worse on nights after a large or late meal. She often has a bad taste in her mouth as she starts coughing. Based on this information, which of the following would be a reasonable empiric therapeutic trial?
A. Inhaled corticosteroid
B. Inhaled long acting beta agonist
C. Nasal corticosteroid
D. Oral proton pump inhibitor
E. Oral triple antibiotic therapy for H. pylori
I-104. A 48-year-old man is evaluated for hypoxia of unknown etiology. He recently has noticed shortness of breath that is worse with exertion and in the upright position. It is relieved with lying down. On physical examination, he is visibly dyspneic with minimal exertion. He is noted to have a resting oxygen saturation of 89% on room air. When lying down, his oxygen saturation increases to 93%. His pulmonary examination shows no wheezes or crackles. His cardiac examination findings are normal without murmur. His chest radiograph reports a possible 1-cm lung nodule in the right lower lobe. On 100% oxygen and in the upright position, the patient has an oxygen saturation of 90%. What is the most likely cause of the patient’s hypoxia?
A. Circulatory hypoxia
B. Hypoventilation
C. Intracardiac right-to-left shunting
D. Intrapulmonary right-to-left shunting
E. Ventilation–perfusion mismatch
I-105. A patient is evaluated in the emergency department for peripheral cyanosis. All of the following are potential etiologies EXCEPT:
A. Cold exposure
B. Deep venous thrombosis
C. Methemoglobinemia
D. Peripheral vascular disease
E. Raynaud’s phenomenon
I-106. An 18-year-old college freshman is being evaluated for a heart murmur heard at health screening. She reports an active lifestyle, no past medical history, and no cardiac symptoms. She has a midsystolic murmur that follows a nonejection sound and crescendos with S2. The murmur duration is greater when going from supine to standing and decreases when squatting. The murmur is heard best along the lower left sternal border and apex. Her electrocardiogram is normal. Which of the following is the most likely condition causing the murmur?
A. Aortic stenosis
B. Hypertrophic obstructive cardiomyopathy
C. Mitral valve prolapse
D. Pulmonic stenosis
E. Tricuspid regurgitation
I-107. Which of the following characteristics makes a heart murmur more likely to be caused by tricuspid regurgitation than mitral regurgitation?
A. Decreased intensity with amyl nitrate
B. Inaudible A2 at the apex
C. Prominent c-v wave in jugular pulse
D. Onset signaled by a midsystolic click
E. Wide splitting of S2
I-108. You are examining a 25-year-old patient in clinic who came in for a routine examination. Cardiac auscultation reveals a second heart sound that is split and does not vary with respiration. There is also a grade 2–3 midsystolic murmur at the midsternal border. Which of the following is most likely?
A. Atrial septal defect
B. Hypertrophic obstructive cardiomyopathy
C. Left bundle branch block
D. Normal physiology
E. Pulmonary hypertension
I-109. A 32-year-old woman presents to her physician complaining of hair loss. She is currently 10 weeks postpartum after delivery of a normal healthy baby girl. She admits to having increased stress and sleep loss because her child has colic. She also has not been able to nurse because of poor milk production. On examination, the patient’s hair does not appear to have decreased density. With a gentle tug, more than 10 hairs come out but are not broken and all appear normal. There are no scalp lesions. What do you recommend for this patient?
A. Careful evaluation of the patient’s hair care products for a potential cause
B. Reassurance only
C. Referral for counseling for trichotillomania
D. Treatment with minoxidil
E. Treatment with topical steroids
I-110. A 26-year-old man develops diffuse itching, wheezing, and laryngeal edema within minutes of receiving intravenous radiocontrast media for an intravenous pyelogram. He has not previously received contrast dye per his recollection. He is treated with supportive care and recovers without further complications. Which of the following best describes the mechanism of the patient’s reaction to the contrast media?
A. Cross-linking of IgE molecules fixed to sensitized cells in the presence of a specific drug-protein conjugate
B. Deposition of circulating immune complexes
C. Development of drug-specific T-cell immunogenicity
D. Direct mast cell degranulation
E. Hepatic metabolism into toxic intermediate
I-111. A 44-year-old woman is prescribed phenytoin for the development of complex partial seizures. One month after initiating the medication, she is evaluated for a diffuse erythematous eruption with associated fever to 101.3°F. She is noted to have facial edema with diffusely enlarged lymph nodes along the cervical, axillary, and inguinal areas. Her white cell count is 14,500/μL (75% neutrophils, 12% lymphocytes, 5% atypical lymphocytes, and 8% eosinophils). A basic metabolic panel is normal, but elevations in the liver functions tests are noted with an AST of 124 U/L, ALT of 148 U/L, alkaline phosphatase of 114 U/L, and total bilirubin of 2.2 mg/dL. All of the following are indicated in the management of this patient EXCEPT:
A. Administration of carbamazepine 200 mg twice daily
B. Administration of prednisone 1.5–2 mg/kg daily
C. Administration of topical glucocorticoids
D. Discontinuation of phenytoin
E. Evaluation for development of thyroiditis for up to 6 months
I-112. Which of the following drugs is associated with development of both phototoxicity and photoallergy?
A. Amiodarone
B. Diclofenac
C. Doxycycline
D. Hydrochlorothiazide
E. Levofloxacin
I-113. You are seeing a patient in follow-up in whom you have begun an evaluation for an elevated hematocrit. You suspect polycythemia vera based on a history of aquagenic pruritus and splenomegaly. Which set of laboratory tests is consistent with the diagnosis of polycythemia vera?
A. Elevated red blood cell mass, high serum erythropoietin levels, and normal oxygen saturation
B. Elevated red blood cell mass, low serum erythropoietin levels, and normal oxygen saturation
C. Normal red blood cell mass, high serum erythropoietin levels, and low arterial oxygen saturation
D. Normal red blood cell mass, low serum erythropoietin levels, and low arterial oxygen saturation
I-114. All of the following are common manifestations of bleeding caused by von Willebrand disease EXCEPT:
A. Angiodysplasia of the small bowel
B. Epistaxis
C. Menorrhagia
D. Postpartum hemorrhage
E. Spontaneous hemarthrosis
I-115. A 68-year-old man is admitted to the intensive care unit with spontaneous retroperitoneal bleeding and hypotension. He has a medical history of hypertension, diabetes mellitus, and chronic kidney disease stage III. His medications include lisinopril, amlodipine, sitagliptin, and glimepiride. On initial presentation, he is in pain and has a blood pressure of 70/40 mmHg with a heart rate of 132 beats/min. His hemoglobin on admission is 5.3 g/dL and hematocrit is 16.0%. His coagulation studies demonstrate an aPTT of 64 seconds and a PT of 12.1 seconds (INR 1.0). Mixing studies (1:1) are performed. Immediately, the aPTT decreases to 42 seconds. At 1 hour, the aPTT is 56 seconds, and at 2 hours, it is 68 seconds. Thrombin time and reptilase time are normal. Fibrinogen is also normal. What is the most likely cause of the patient’s coagulopathy?
A. Acquired factor VIII deficiency
B. Acquired factor VIII inhibitor
C. Heparin
D. Lupus anticoagulant
E. Vitamin K deficiency
I-116. A 54-year-old man is seen in the clinic complaining of painless enlargement of lymph nodes in his neck. He has not otherwise been ill and denies fevers, chills, weight loss, and fatigue. His past medical history is remarkable for pulmonary tuberculosis that was treated 10 years previously under directly observed therapy. He currently takes no medications. He is a heterosexual man in a monogamous relationship for 25 years. He denies illicit drug use. He has smoked 1½ packs of cigarettes daily since 16 years of age. He works as a logger. On physical examination, the patient is thin, but not ill-appearing. He is not febrile and has normal vital signs. He has dental caries noted with gingivitis. In the right supraclavicular area, there is a hard and fixed lymph node measuring 2.5 × 2.0 cm in size. Lymph nodes less than 1 cm in size are noted in the anterior cervical chain. There is no axillary or inguinal lymphadenopathy. His liver and spleen are not enlarged. Which of the following factors in history or physical examination increases the likelihood that the lymph node enlargement is caused by malignancy?
A. Age greater than 50 years
B. Location in the supraclavicular area
C. Presence of a lymph node that is hard and fixed
D. Size greater than 2.25 cm2 (1.5 × 1.5 cm)
E. All of the above
I-117. A 24-year-old woman presents for a routine checkup and complains only of small masses in her groin. She states that they have been present for at least 3 years. She denies fever, malaise, weight loss, and anorexia. She works as a sailing instructor and competes in triathlons. On physical examination, she is noted to have several palpable 1-cm inguinal lymph nodes that are mobile, non-tender, and discrete. There is no other lymphadenopathy or focal findings on examination. What should be the next step in management?
A. Bone marrow biopsy
B. CT scan of the chest, abdomen, and pelvis
C. Excisional biopsy
D. Fine-needle aspiration for culture and cytopathology
E. Pelvic ultrasonography
F. Reassurance
I-118. All of the following diseases are associated with massive splenomegaly (spleen extends 8 cm below the costal margin or weighs >1000 g) EXCEPT:
A. Autoimmune hemolytic anemia
B. Chronic lymphocytic leukemia
C. Cirrhosis with portal hypertension
D. Marginal zone lymphoma
E. Myelofibrosis with myeloid metaplasia
I-119. The presence of Howell-Jolly bodies, Heinz bodies, basophilic stippling, and nucleated red blood cells in a patient with hairy cell leukemia before any treatment intervention implies which of the following?
A. Diffuse splenic infiltration by tumor
B. Disseminated intravascular coagulation (DIC)
C. Hemolytic anemia
D. Pancytopenia
E. Transformation to acute leukemia
I-120. Which of the following is true regarding infection risk after elective splenectomy?
A. Patients are at no increased risk of viral infection after splenectomy.
B. Patients should be vaccinated 2 weeks after splenectomy.
C. Splenectomy patients over the age of 50 are at greatest risk for postsplenectomy sepsis.
D. Staphylococcus aureus is the most commonly implicated organism in postsplenectomy sepsis.
E. The risk of infection after splenectomy increases with time.
I-121. An 18-year-old man is seen in consultation for a pulmonary abscess caused by infection with Staphylococcus aureus. He had been in his usual state of health until 1 week ago when he developed fevers and a cough. He has no ill contacts and presents in the summer. His medical history is significant for episodes of axillary and perianal abscesses requiring incision and drainage. He cannot specifically recall how often this has occurred, but he does know it has been more than five times that he can recall. In one instance, he recalls a lymph node became enlarged to the point that it “popped” and drained spontaneously. He also reports frequent aphthous ulcers and is treated for eczema. On physical examination, his height is 5′3′′. He appears ill with a temperature of 39.6°C. Eczematous dermatitis is present in the scalp and periorbital area. There are crackles at the left lung base. Axillary lymphadenopathy is present bilaterally and is tender. The spleen in enlarged. His laboratory studies show a white blood cell count of 12,500/μL (94% neutrophils), hemoglobin of 11.3 g/dL, hematocrit of 34.2%, and platelets of 320,000/μL. Granulomatous inflammation is seen on lymph node biopsy. Which of the following tests are most likely found in this patient?
A. Elevated angiotensin-converting enzyme level
B. Eosinophilia
C. Giant primary granules in neutrophils
D. Mutations of the tumor necrosis factor-alpha receptor
E. Positive nitroblue tetrazolium dye test
I-122. A 72-year-old man with chronic obstructive pulmonary disease and stable coronary disease presents to the emergency department with several days of worsening productive cough, fevers, malaise, and diffuse muscle aches. A chest radiograph demonstrates a new lobar infiltrate. Laboratory measurements reveal a total white blood cell count of 12,100 cells/μL with a neutrophilic predominance of 86% and 8% band forms. He is diagnosed with community-acquired pneumonia, and antibiotic treatment is initiated. Under normal, or “nonstress,” conditions, what percentage of the total body neutrophils are present in the circulation?
A. 2%
B. 10%
C. 25%
D. 40%
E. 90%
I-123. A patient with longstanding HIV infection, alcoholism, and asthma is seen in the emergency department for 1–2 days of severe wheezing. He has not been taking any medicines for months. He is admitted to the hospital and treated with nebulized therapy and systemic glucocorticoids. His CD4 count is 8 and viral load is greater than 750,000. His total white blood cell (WBC) count is 3200 cells/μL with 90% neutrophils. He is accepted into an inpatient substance abuse rehabilitation program and before discharge is started on opportunistic infection prophylaxis, bronchodilators, a prednisone taper over 2 weeks, ranitidine, and highly active antiretroviral therapy. The rehabilitation center pages you 2 weeks later; a routine laboratory check reveals a total WBC count of 900 cells/μL with 5% neutrophils. Which of the following new drugs would most likely explain this patient’s neutropenia?
A. Darunavir
B. Efavirenz
C. Ranitidine
D. Prednisone
E. Trimethoprim–sulfamethoxazole
I-124. All of the following statements regarding mercury exposure or poisoning are true EXCEPT:
A. Chronic mercury poisoning is best assessed using hair samples.
B. Ethyl mercury preservative in multiuse vaccines has not been implicated in causing autism.
C. Exposure to as little as a few drops of dimethylmercury may be lethal.
D. Offspring of mothers who ingested mercury-contaminated fish are at higher risk of neurobehavioral abnormalities.
E. Pregnant women should avoid consumption of sardines and mackerel.
I-125. A 39-year-old man comes to clinic reporting a 4-day illness that began while he was in the Caribbean on vacation. A few hours after attending a large seafood buffet, he developed abdominal pain, chills, nausea, and diarrhea. Soon thereafter, he noticed diffused paresthesias, throat numbness, and fatigue. The symptoms slowly improved over 2 days, and he returned home yesterday. Today he noticed while washing that cold water felt hot and warm water felt cold. He is concerned about this new symptom. All of the following are true regarding his illness EXCEPT:
A. His symptoms should improve over weeks to months.
B. It is likely caused by ingestion of contaminated snapper or grouper.
C. It is likely caused by ingestion of undercooked oysters or clams.
D. Subsequent episodes may be more severe.
E. No diagnostic laboratory test is available.
I-126. Which of the following is the most common cause of death from poisoning?
A. Acetaminophen
B. Carbon monoxide
C. Chlorine gas
D. Insecticide
E. Tricyclic antidepressants
I-127. Which of the following is a distinguishing feature of amphetamine overdose versus other causes of sympathetic overstimulation caused by drug overdose or withdrawal?
A. Hallucination
B. Hot, dry, flushed skin and urinary retention
C. History of benzodiazepine abuse
D. Markedly increased blood pressure, heart rate, and end-organ damage in the absence of hallucination
E. Nystagmus
I-128. A patient with metabolic acidosis, reduced anion gap, and increased osmolal gap is most likely to have which of the following toxic ingestions?
A. Lithium
B. Methanol
C. Oxycodone
D. Propylene glycol
E. Salicylate
I-129. Which of the following is true regarding drug effects after an overdose compared with a reference dose?
A. Drug effects begin earlier, peak earlier, and last longer.
B. Drug effects begin earlier, peak later, and last longer.
C. Drug effects begin earlier, peak later, and last shorter.
D. Drug effects begin later, peak earlier, and last shorter.
E. Drug effects begin later, peak later, and last longer.
I-130. Which of the following statements regarding gastric decontamination for toxin ingestion is true?
A. Activated charcoal’s most common side effect is aspiration.
B. Gastric lavage via nasogastric tube is preferred over the use of activated charcoal when therapeutic endoscopy may also be warranted.
C. Syrup of ipecac has no role in the hospital setting.
D. There are insufficient data to support or exclude a benefit when gastric decontamination is used more than 1 hour after a toxic ingestion.
E. All of the above are true.
I-131. One of your patients is contemplating a trekking trip to Nepal at elevations between 2500 and 3000 m. Five years ago, while skiing at Telluride (altitude, 2650 m), she recalls having headache, nausea, and fatigue within 1 day of arriving that lasted about 2–3 days. All of the following are true regarding the development of acute mountain sickness in this patient EXCEPT:
A. Acetazolamide starting 1 day before ascent is effective in decreasing the risk.
B. Gingko biloba is not effective in decreasing the risk.
C. Gradual ascent is protective.
D. Her prior episode increases her risk for this trip.
E. Improved physical conditioning before the trip decreases the risk.
I-132. A 36-year-old man develops shortness of breath, dyspnea, and dry cough 3 days after arriving for helicopter snowboarding in the Bugaboo mountain range in British Columbia (elevation, 3000 m). Over the next 12 hours, he becomes more short of breath and produces pink, frothy sputum. An EMT-trained guide hears crackles on chest examination. All of the following are true regarding his illness EXCEPT:
A. Descent and oxygen are most therapeutic.
B. Exercise increased his risk.
C. Fever and leukocytosis may occur.
D. He should never risk return to high altitude after recovery.
E. Pretreatment with nifedipine or tadalafil would have lowered his risk.
I-133. Which of the following is considered an absolute contraindication to hyperbaric oxygen therapy?
A. Carbon monoxide poisoning
B. History of COPD
C. History of high altitude pulmonary edema
D. Radiation proctitis
E. Untreated pneumothorax
I-134. A 35-year-old woman is scuba diving while vacationing in Malaysia. During her last dive of the day, her regulator malfunctions, requiring her to ascend from 20 m to the surface rapidly. Upon returning to the boat, she feels well. However, about 6 hours after returning to shore, she develops diffuse itching and muscle aches, leg pain, blurred vision, slurred speech, and nausea. Which of the following statements regarding her condition is true?
A. Decompression illness is unlikely at 20-m water depth.
B. Inhalation of 100% oxygen is contraindicated.
C. She can never again scuba dive to a depth greater than 6 m.
D. She should receive recompression and hyperbaric oxygen therapy.
E. She should remain upright as much as possible.
I-135. Which of the following statements regarding the distinction between acute lung injury (ALI) and acute respiratory distress syndrome (ARDS) is true?
A. ALI and ARDS can be distinguished by radiographic testing.
B. ALI and ARDS can be distinguished by the magnitude of the PaO2/FIO2 ratio.
C. ALI can be diagnosed in the presence of elevated left atrial pressure, but ARDS can not.
D. ALI is caused by direct lung injury, but ARDS is the result of secondary lung injury.
E. The risk of ALI but not ARDS increases with multiple predisposing conditions.
I-136. Which of the following has been demonstrated to reduce mortality in patients with ARDS?
A. High-dose glucocorticoids within 48 hours of presentation
B. High-frequency mechanical ventilation
C. Inhaled nitric oxide
D. Low tidal volume mechanical ventilation
E. Surfactant replacement
I-137. A 38-year-old man is hospitalized in the ICU with ARDS after a motor vehicle accident with multiple long bone fractures, substantial blood loss, and hypotension. By day 2 of hospitalization, he is off vasopressors but is requiring a high FIO2 and positive end-expiratory pressure (PEEP) to maintain adequate oxygenation. His family is asking about the short- and long-term prognosis for recovery. All of the following statements about his prognosis are true EXCEPT:
A. He has a greater chance of survival than a patient with similar physiology who is older than 70 years old.
B. His overall mortality from ARDS is approximately 25–45%.
C. If he survives, he is likely to have some degree of depression or posttraumatic stress disorder.
D. If he survives, he likely will have normal or near normal lung function.
E. The most likely cause of mortality is hypoxemic respiratory failure.
I-138. Clinical trials support the use of noninvasive ventilation in which of the following patients?
A. A 33-year-old man who was rescued from a motor vehicle accident. He is unarousable with possible internal injuries. Room air blood gas is 7.30 (pH), PCO2 50 mmHg, PO2 60 mmHg.
B. A 49-year-old woman with end-stage renal disease admitted with presumed staphylococcal sepsis from her hemodialysis catheter. She is somnolent, blood pressure is 80/50 mmHg, heart rate is 105 beats/min, and room air oxygen saturation is 95%.
C. A 58-year-old woman with a history of cirrhotic liver disease admitted with a presumed esophageal variceal bleed. Her blood pressure is 75/55 mmHg, and she has a heart rate of 110 beats/min. She is awake and alert.
D. A 62-year-old man with a long history of COPD admitted with an exacerbation related to an upper respiratory tract infection. He is in marked respiratory distress but is awake and alert. Chest radiograph only shows hyperinflation. His room air arterial blood gas is pH, 7.28; PCO2, 75 mmHg; and PO2, 46 mmHg.
E. A 74-year-old man with cardiogenic shock and an acute ST-segment elevation myocardial infarction. His blood pressure is 84/65 mmHg, heart rate is 110 beats/min, respiratory rate is 24 breaths/min, and room air oxygen saturation is 85%.
I-139. You are caring for a patient on mechanical ventilation in the intensive care unit. Whenever the patient initiates a breath, no matter her spontaneous respiratory rate, she gets a fixed volume breath from the machine that does not change from breath to breath. After receiving a dose of sedation, she does not initiate any breaths, but the machine delivers the same volume breath at periodic fixed intervals during this time. Which of the following modes of mechanical ventilation is this patient receiving?
A. Assist control
B. Continuous positive airway pressure
C. Pressure control
D. Pressure support
E. Synchronized intermittent mandatory ventilation (SIMV)
I-140. A 68-year-old woman has been receiving mechanical ventilation for 10 days for community-acquired pneumonia. You are attempting to decide whether the patient is appropriate for a spontaneous breathing trial. All of the following factors would indicate that the patient is likely to be successfully extubated EXCEPT:
A. Alert mental status
B. PEEP of 5 cmH2O
C. pH greater than 7.35
D. Rapid shallow breathing index (respiratory rate/tidal volume) greater than 105
E. SaO2 greater than 90% and FIO2 less than 0.5
I-141. A 45-year-old woman with HIV is admitted to the intensive care unit with pneumonia and pneumothorax secondary to infection with Pneumocystis jiroveci. She requires mechanical ventilatory support, chest tube placement, and central venous access. The ventilator settings are PC mode; inspiratory pressure, 30 cmH2O, 1.0; and PEEP, 10 cmH2O. An arterial blood gas measured on these settings shows: pH 7.32, 46 mmHg, and 62 mmHg. All of the following are important supportive measures for this patient EXCEPT:
A. Analgesia to maintain patient comfort
B. Daily change of ventilator circuit
C. Gastric acid suppression
D. Nutritional support
E. Prophylaxis against deep venous thrombosis
I-142. All of the following statements about the physiology of mechanical ventilation are true EXCEPT:
A. Application of PEEP decreases left ventricular preload and afterload.
B. High inspired tidal volumes contribute to the development of acute lung injury caused by overdistention of alveoli with resultant alveolar damage.
C. Increasing the inspiratory flow rate will decrease the ratio of inspiration to expiration (I:E) and allow more time for expiration.
D. Mechanical ventilation provides assistance with inspiration and expiration.
E. PEEP helps prevent alveolar collapse at end-expiration.
I-143. A 64-year-old man requires endotracheal intubation and mechanical ventilation for chronic obstructive pulmonary disease. He was paralyzed with rocuronium for intubation. His initial ventilator settings were AC mode; respiratory rate 10 breaths/minute; FIO2 1.0; Vt (tidal volume) 550 mL; and positive end-expiratory pressure 0 cm H2O. On admission to the intensive care unit the patient remains paralzyed; arterial blood gas is pH 7.22, PCO2 78 mmHg, PO2 394 mmHg. The FIO2 is decreased to 0.6. Thirty minutes later you are called to the bedside to evaluate the patient for hypotension. Current vital signs are blood pressure 80/40 mmHg, heart rate, 133 beats/min; respiratory rate, 24/minute; and oxygen saturation 92%. Physical examination shows the patient is agitated and moving all extremities, a prolonged expiration with wheezing continuing until the initiation of the next breath. Breath sounds are heard in both lung fields. The high-pressure alarm on the ventilator is triggering. What should be done first in treating this patient’s hypotension?
A. Administer a fluid bolus of 500 mL.
B. Disconnect the patient from the ventilator.
C. Initiate a continuous IV infusion of midazolam.
D. Initiate a continuous IV infusion of norepinephrine.
E. Perform tube thoracostomy on the right side.
I-144. All of the following are relative contraindications for the use of succinylcholine as a paralytic for endotracheal intubation EXCEPT:
A. Acetaminophen overdose
B. Acute renal failure
C. Crush injuries
D. Muscular dystrophy
E. Tumor lysis syndrome
I-145. Match the following vasopressors with the statement that best describes their action on the cardiovascular system.
1. Dobutamine
2. Low-dose dopamine (2–4 μg/kg/min)
3. Norepinephrine
4. Phenylephrine
A. Acts solely at α-adrenergic receptors to cause vasoconstriction
B. Acts at β1-adrenergic receptors and dopaminergic receptors to increase cardiac contractility and heart rate; also causes vasodilatation and increased splanchnic and renal blood flow
C. Acts at β1- and, to a lesser extent, β2-adrenergic receptors to increase cardiac contractility, heart rate, and vasodilatation
D. Acts at α- and β1-adrenergic receptors to increase heart rate, cardiac contractility, and vasoconstriction
I-146. An 86-year-old nursing home resident is brought by ambulance to the local emergency department. He was found unresponsive in his bed immersed in black stool. Apparently, he had not been feeling well for 1–2 days, had complained of vague abdominal pain, and had decreased oral intake; no further history is available from the nursing home staff. His past medical history is remarkable for Alzheimer’s dementia and treated prostate cancer. The emergency responders were able to appreciate a faint pulse and obtained a blood pressure of 91/49 mmHg and a heart rate of 120 beats/min. In the emergency department, his pressure is 88/51 mmHg and heart rate is 131 beats/min. He is moaning and obtunded, localizes to pain, and has flat neck veins. Skin tenting is noted. A central venous catheter is placed that reveals CVP less than 5 mmHg, specimens for initial laboratory testing are sent off, and electrocardiogram and chest x-ray are obtained. Catheterization of the bladder yields no urine. Anesthesiology has been called to the bedside and is assessing the patient’s airway. What is the best immediate step in management?
A. Infuse hypertonic saline to increase the rate of vascular filling.
B. Infuse isotonic crystalloid solution via IV wide open.
C. Infuse a colloidal solution rapidly.
D. Initiate inotropic support with dobutamine.
E. Initiate IV pressors starting with Levophed.
I-147. In the patient described above, which of the following is true regarding his clinical condition?
A. Loss of 20–40% of the blood volume leads to shock physiology.
B. Loss of less than 20% of the blood volume will manifest as orthostasis.
C. Oliguria is a crucial prognostic sign of impending vascular collapse.
D. Symptoms of hypovolemic shock differ from those of hemorrhagic shock.
E. The first sign of hypovolemic shock is mental obtundation.
I-148. A 52-year-old man presents with crushing substernal chest pain. He has a history of coronary artery disease and has had two non–ST-segment elevation myocardial infarctions in the past 5 years, both requiring percutaneous intervention and intracoronary stent placement. His electrocardiogram shows ST elevations across the precordial leads, and he is taken emergently to the catheterization laboratory. After angioplasty and stent placement, he is transferred to the coronary care unit. His vital signs are stable on transfer; however, 20 minutes after arrival, he is found to be unresponsive. His radial pulse is thready, extremities are cool, and blood pressure is difficult to obtain; with a manual cuff, it is 65/40 mmHg. The nurse turns to you and asks what you would like to do next. Which of the following accurately represents the physiologic characteristics of this patient’s condition?
I-149. All of the following are factors that are related to the increased incidence of sepsis in the United States EXCEPT:
A. Aging of the population
B. Increased longevity of individuals with chronic disease
C. Increased risk of sepsis in individuals without comorbidities
D. Increased risk of sepsis in individuals with AIDS
E. Increased use of immunosuppressive drugs
I-150. A 68-year-old woman is brought to the emergency department for fever and lethargy. She first felt ill yesterday and experienced generalized body aches. Overnight, she developed a fever of 39.6°C and had shaking chills. By this morning, she was feeling very fatigued. Her son feels that she has had periods of waxing and waning mental status. She denies cough, nausea, vomiting, diarrhea, and abdominal pain. She has a medical history of rheumatoid arthritis. She takes prednisone, 10 mg daily, and methotrexate, 15 mg weekly. On examination, she is lethargic but appropriate. Her vital signs are blood pressure of 85/50 mmHg, heart rate of 122 beats/min, temperature of 39.1°C, respiratory rate of 24 breaths/min, and oxygen saturation of 97% on room air. Physical examination shows clear lung fields and a regular tachycardia without murmur. There is no abdominal tenderness or masses. Stool is negative for occult blood. There are no rashes. Hematologic studies show a white blood cell count of 24,200/μL with a differential of 82% PMNs, 8% band forms, 6% lymphocytes, and 3% monocytes. Hemoglobin is 8.2 g/dL. A urinalysis has numerous white blood cells with gram-negative bacteria on Gram stain. Chemistries reveal the following: bicarbonate of 16 meq/L, BUN of 60 mg/dL, and creatinine of 2.4 mg/dL. After fluid administration of 2 L, the patient has a blood pressure of 88/54 mmHg and a heart rate of 112 beats/min with a central venous pressure of 18 cmH2O. There is 25 mL of urine output in the first hour. The patient has been initiated on antibiotics with cefepime. What should be done next for the treatment of this patient’s hypotension?
A. Dopamine, 3 μg/kg/min IV
B. Hydrocortisone, 50 mg IV every 6 hours
C. Norepinephrine, 2 μg/min IV
D. Ongoing colloid administration at 500–1000 mL/h
E. Transfusion of 2 units of packed red blood cells
I-151. All of the following statements about the pathogenesis of sepsis and septic shock are true EXCEPT:
A. Blood cultures are positive in only 20–40% of cases of severe sepsis.
B. Microbial invasion of the bloodstream is not necessary for the development of severe sepsis.
C. Serum levels of TNF-alpha are typically reduced in patients with severe sepsis or septic shock.
D. The hallmark of septic shock is a marked decrease in peripheral vascular resistance that occurs despite increased plasma levels of catecholamines.
E. Widespread vascular endothelial injury is present in severe sepsis and is mediated by cytokines and procoagulant factors that stimulate intravascular thrombosis.
I-152. Which of the following treatments is recommended to improve mortality in septic shock?
A. Activated protein C (drotrecogin alpha)
B. Administration of antibiotics within 1 hour of presentation
C. Bicarbonate therapy for severe acidosis
D. Erythropoietin
E. Vasopressin infusion
I-153. All of the following statements regarding cardiogenic shock are true EXCEPT:
A. Approximately 80% of cases of cardiogenic shock complicating acute myocardial infarction are attributable to acute severe mitral regurgitation.
B. Cardiogenic shock is more common in ST-segment elevation than non–ST-segment elevation myocardial infarction.
C. Cardiogenic shock is uncommon in inferior wall myocardial infarction.
D. Cardiogenic shock may occur in the absence of significant coronary stenosis.
E. Pulmonary capillary wedge pressure is elevated in cardiogenic shock.
I-154. Aortic counterpulsation with an intra-aortic balloon pump has which of the following as an advantage over therapy with infused vasopressors or inotropes in a patient with acute ST-segment elevation myocardial infarction and cardiogenic shock?
A. Increased heart rate
B. Increased left ventricular afterload
C. Lower diastolic blood pressure
D. Not contraindicated in acute aortic regurgitation
E. Reduced myocardial oxygen consumption
I-155. Which of the following is the most common electrical mechanism to explain sudden cardiac death?
A. Asystole
B. Bradycardia
C. Pulseless electrical activity (PEA)
D. Pulseless ventricular tachycardia (PVT)
E. Ventricular fibrillation
I-156. All of the following statements regarding successful resuscitation from sudden cardiac death are true EXCEPT:
A. Advanced age does not affect the likelihood of immediate resuscitation, only the probability of hospital discharge.
B. After cardiac out of hospital cardiac arrest, survival rates are approximately 25% if defibrillation is administered after 5 minutes.
C. If the initial rhythm in an out-of-hospital cardiac arrest is pulseless ventricular tachycardia, the patient has a higher probability of survival than asystole.
D. Prompt CPR followed by prompt defibrillation improves outcomes in all settings.
E. The probability of survival from cardiac arrest is higher if the event takes place in a public setting than at home.
I-157. A 28-year-old woman has severe head trauma after a motor vehicle accident. One year after the accident, she is noted to have spontaneous eye opening and is able to track an object visually at times. She does not speak or follow any commands. She breathes independently but is fed through a gastrostomy tube. She can move all extremities spontaneously but without purposeful movement. What term best describes this patient’s condition?
A. Coma
B. Locked-in
C. Minimally conscious state
D. Persistent vegetative state
E. Vegetative state
I-158. A 52-year-old man is evaluated after a large subarachnoid hemorrhage (SAH) from a ruptured cerebral aneurysm. There is concern that the patient has brain death. What test is most commonly used to diagnose brain death in this situation?
A. Apnea testing
B. Cerebral angiography
C. Demonstration of absent cranial nerve reflexes
D. Demonstration of fixed and dilated pupils
E. Performance of transcranial Doppler ultrasonography
I-159. Which of the following neurologic phenomena is classically associated with herniation of the brain through the foramen magnum?
A. Third-nerve compression and ipsilateral papillary dilation
B. Catatonia
C. “Locked-in” state
D. Miotic pupils
E. Respiratory arrest
I-160. A 72-year-old woman is admitted to the intensive care unit after a cardiac arrest at home. She had a witnessed collapse, and her family immediately began to perform cardiopulmonary resuscitation. Emergency medical service arrived within 10 minutes, and the initial cardiac rhythm demonstrated ventricular fibrillation. Spontaneous circulation returned after defibrillation, and the estimated time the patient was without a pulse was 15–20 minutes. The patient is brought to hospital and remains intubated, paralyzed, and sedated in the coronary care unit. She is being treated with medically induced hypothermia and is completely unresponsive to all stimuli 12 hours after the initial event. Her pupils are 3 mm and respond sluggishly to light. She has no cough or gag reflex. Intermittent myoclonic jerks are seen. The family has concerns about her neurologic prognosis after her prolonged cardiac arrest. What advice do you give the family regarding prognosis in this situation?
A. An MRI scan of the brain should be performed before determining neurologic outcome.
B. Apnea testing will be performed at the first opportunity to determine if the patient has suffered brain death.
C. Given the immediate actions of the family to initiate cardiopulmonary resuscitation, the patient has a greater than 50% chance to have good neurologic outcomes.
D. It is impossible to predict the patient’s likelihood of neurologic recovery as her examination is unreliable in the face of sedation and hypothermia.
E. No information regarding prognosis can be determined until 72 hours have passed.
I-161. A 52-year-old man presents to the emergency department complaining of the worst headache of his life that is unresolving. It began abruptly 3 days before presentation and is worse with bending over. It rapidly increased in intensity over 30 minutes, but he did not seek medical care at that time. Over the ensuing 72 hours, the headache has persisted although lessened in intensity. He has not lost consciousness and has no other neurologic symptoms. His vision is normal, but he does report that light is painful to his eyes. His past medical history is notable for hypertension, but he takes his medications irregularly. Upon arrival to the emergency department, his initial blood pressure is 232/128 mmHg with a heart rate of 112 beats/min. No nuchal rigidity is present. A head CT shows no acute bleeding and no mass effect. What is the next best step in the management of this patient?
A. Cerebral angiography
B. CT angiography
C. Lumbar puncture
D. Magnetic resonance angiography
E. Treat with sumatriptan
I-162. A 56-year-old man is admitted to intensive care with a subarachnoid hemorrhage. Upon admission, he is unresponsive, and his head CT shows evidence of blood in the third ventricle with midline shift. He undergoes successful coiling of an aneurysm of the anterior cerebral artery. All of the following would be indicated in the management of this patient EXCEPT:
A. Glucocorticoids
B. Hypernatremia
C. Nimodipine
D. Ventriculostomy
E. Volume expansion
I-163. A 56-year-old man is admitted to the intensive care unit with a hypertensive crisis after cocaine use. Initial blood pressure is 245/132 mmHg. On physical examination, the patient is unresponsive except to painful stimuli. He has been intubated for airway protection and is being mechanically ventilated, with a respiratory rate of 14 breaths/min. His pupils are reactive to light, and he has normal corneal, cough, and gag reflexes. The patient has a dense left hemiparesis. When presented with painful stimuli, the patient responds with flexure posturing on the right side. Computed tomography (CT) reveals a large area of intracranial bleeding in the right frontoparietal area. Over the next several hours, the patient deteriorates. The most recent examination reveals a blood pressure of 189/100 mmHg. The patient now has a dilated pupil on the right side. The patient continues to have corneal reflexes. You suspect rising intracranial pressure related to the intracranial bleed. All but which of the following can be done to decrease the patient’s intracranial pressure?
A. Administer intravenous mannitol at a dose of 1 g/kg body weight.
B. Administer hypertonic fluids to achieve a goal sodium level of 155–160 meq/L.
C. Consult neurosurgery for an urgent ventriculostomy.
D. Initiate intravenous nitroprusside to decrease the mean arterial pressure (MAP) to a goal of 100 mmHg.
E. Increase the respiratory rate to 30 breaths/min.
I-164. A 64-year-old man presents to the emergency department complaining of shortness of breath and facial swelling. He smokes 1 pack of cigarettes daily and has done so since the age of 16 years. On physical examination, he has dyspnea at an angle of 45 degrees or less. His vital signs are heart rate of 124 beats/min, blood pressure of 164/98 mmHg, respiratory rate of 28 breaths/min, temperature of 37.6°C (99.6°F), and oxygen saturation of 89% on room air. Pulsus paradoxus is not present. His neck veins are dilated and do not collapse with inspiration. Collateral venous dilation is noted on the upper chest wall. There is facial edema and 1+ edema of the upper extremities bilaterally. Cyanosis is present. There is dullness to percussion and decreased breath sounds over the lower half of the right lung field. Given this clinical scenario, what would be the most likely finding on CT examination of the chest?
A. A central mass lesion obstructing the right mainstem bronchus
B. A large apical mass invading the chest wall and brachial plexus
C. A large pericardial effusion
D. A massive pleural effusion leading to opacification of the right hemithorax
E. Enlarged mediastinal lymph nodes causing obstruction of the superior vena cava
I-165. In the scenario in question I-165, the initial therapy of this patient includes all of the following EXCEPT:
A. Administration of furosemide as needed to achieve diuresis
B. Elevation of the head of the bed to 45 degrees
C. Emergent radiation
D. Low-sodium diet
E. Oxygen
I-166. A 58-year-old woman with known stage IV breast cancer presents to the emergency department with an inability to move her legs. She has had lower back pain for the past 4 days and has found it difficult to lie down. There is no radiating pain. Earlier today, the patient lost the ability to move either of her legs. In addition, she has been incontinent of urine recently. She has been diagnosed previously with metastatic disease to the lung and pleura from her breast cancer but was not known to have spinal or brain metastases. Her physical examination confirms absence of movement in the bilateral lower extremities associated with decreased to absent sensation below the umbilicus. There is increased tone and 3+ deep tendon reflexes in the lower extremities with crossed adduction. Anal sphincter tone is decreased, and the anal wink reflex is absent. What is the most important first step to take in the management of this patient?
A. Administer dexamethasone 10 mg intravenously.
B. Consult neurosurgery for emergent spinal decompression.
C. Consult radiation oncology for emergent spinal radiation.
D. Perform MRI of the brain.
E. Perform MRI of the entire spinal cord.
I-167. A 21-year-old man is treated with induction chemotherapy for acute lymphoblastic leukemia. His initial white blood cell count before treatment was 156,000/μL. All of the following are expected complications during his treatment EXCEPT:
A. Acute kidney injury
B. Hypercalcemia
C. Hyperkalemia
D. Hyperphosphatemia
E. Hyperuricemia
I-168. All of the following would be important for prevention of these complications EXCEPT:
A. Administration of allopurinol 300 mg/m2 daily
B. Administration of intravenous fluids at a minimum of 3000 mL/m2 daily
C. Alkalinization of the urine to a pH of greater than 7.0 by administration of sodium bicarbonate
D. Frequent monitoring of serum chemistries every 4 hours
E. Prophylactic hemodialysis before initiating chemotherapy